Menu Close

if-f-z-k-1-n-a-k-z-k-a-k-z-C-Prove-a-k-1-2pii-z-r-f-z-z-k-1-dz-




Question Number 61755 by arcana last updated on 08/Jun/19
if f(z)=Σ_(k=1) ^n a_k z^k ,a_k ,z∈C.Prove    a_k =(1/(2πi))∫_(∣z∣=r ) ((f(z))/z^(k+1) )dz
$$\mathrm{if}\:{f}\left({z}\right)=\underset{{k}=\mathrm{1}} {\overset{{n}} {\sum}}{a}_{{k}} {z}^{{k}} ,{a}_{{k}} ,{z}\in\mathbb{C}.\mathrm{Prove} \\ $$$$ \\ $$$${a}_{{k}} =\frac{\mathrm{1}}{\mathrm{2}\pi{i}}\underset{\mid{z}\mid={r}\:} {\int}\frac{{f}\left({z}\right)}{{z}^{{k}+\mathrm{1}} }{dz} \\ $$$$ \\ $$
Commented by arcana last updated on 08/Jun/19
use ∫_(∣z∣=r) (1/z^n )dz=2πi iff n=1,∫_(∣z∣=r) (1/z^n )dz=0 if n≠1
$$\mathrm{use}\:\underset{\mid{z}\mid={r}} {\int}\frac{\mathrm{1}}{{z}^{{n}} }{dz}=\mathrm{2}\pi{i}\:\mathrm{iff}\:{n}=\mathrm{1},\underset{\mid{z}\mid={r}} {\int}\frac{\mathrm{1}}{{z}^{{n}} }{dz}=\mathrm{0}\:\mathrm{if}\:{n}\neq\mathrm{1} \\ $$
Commented by arcana last updated on 09/Jun/19
Cauchy theorem  ((k!)/(2πi))∫_C_R  ((f(z))/z^(k+1) )dz=f^( (k)) (0)?
$${Cauchy}\:{theorem} \\ $$$$\frac{{k}!}{\mathrm{2}\pi{i}}\underset{{C}_{{R}} } {\int}\frac{{f}\left({z}\right)}{{z}^{{k}+\mathrm{1}} }{dz}={f}^{\:\left({k}\right)} \left(\mathrm{0}\right)? \\ $$
Answered by perlman last updated on 09/Jun/19
cauchy theorem∫_(cr) ((f(z))/z^(k+1) )=(1/((k)!))f^((k)) (0)=(1/(k!))(d^k x/dx^k )Σ_(k=1) (a_k z^k )=(1/(k!))Σ_(j≥k) a_k d^k x^j =(1/(k!))Σ_(j≥k) a_j j(j−1)......(j−k+1)x^(j−k) ∣_(x=0)   =(1/(k!))a_k k(k−1).....1+(1/(k!))Σ_(j>k) a_j j(j−1)....(j−k+1)x^(j−k) ∣_(x=0) =a_k ((k!)/(k!))+0  =a_k
$${cauchy}\:{theorem}\int_{{cr}} \frac{{f}\left({z}\right)}{{z}^{{k}+\mathrm{1}} }=\frac{\mathrm{1}}{\left({k}\right)!}{f}^{\left({k}\right)} \left(\mathrm{0}\right)=\frac{\mathrm{1}}{{k}!}\frac{{d}^{{k}} {x}}{{dx}^{{k}} }\underset{{k}=\mathrm{1}} {\sum}\left({a}_{{k}} {z}^{{k}} \right)=\frac{\mathrm{1}}{{k}!}\underset{{j}\geqslant{k}} {\sum}{a}_{{k}} {d}^{{k}} {x}^{{j}} =\frac{\mathrm{1}}{{k}!}\sum_{{j}\geqslant{k}} {a}_{{j}} {j}\left({j}−\mathrm{1}\right)……\left({j}−{k}+\mathrm{1}\right){x}^{{j}−{k}} \mid_{{x}=\mathrm{0}} \\ $$$$=\frac{\mathrm{1}}{{k}!}{a}_{{k}} {k}\left({k}−\mathrm{1}\right)…..\mathrm{1}+\frac{\mathrm{1}}{{k}!}\underset{{j}>{k}} {\sum}{a}_{{j}} {j}\left({j}−\mathrm{1}\right)….\left({j}−{k}+\mathrm{1}\right){x}^{{j}−{k}} \mid_{{x}=\mathrm{0}} ={a}_{{k}} \frac{{k}!}{{k}!}+\mathrm{0} \\ $$$$={a}_{{k}} \\ $$

Leave a Reply

Your email address will not be published. Required fields are marked *